HELP PLEASE
Please find the angle measure . Do not use the degree symbol.

Please classify as acute, obtuse, right, or straight . No capital letters please.

HELP PLEASE Please Find The Angle Measure . Do Not Use The Degree Symbol.Please Classify As Acute, Obtuse,

Answers

Answer 1

Answer:

acute

Step-by-step explanation:


Related Questions

Need help answering question 4

Answers

Answer:

(-1,5)

Step-by-step explanation:

To find this, you have to add both "x" coordinates, then divide them by two.

Same thing goes for the "y" coordinates, add them, then divide them by two.

x-coordinates --->  2+(-4)= -2

y-coordinates ---> 1+9= 10

-2÷2= -1

10÷2= 5

That gives you, (-1,5).

Please answer the question and give an explanation. Will give brainliest.

Answers

Step-by-step explanation:

By Pythagoras' Theorem, z^2 + (2.7)^2 = (4.2)^2.

This means that z^2 = (4.2)^2 - (2.7)^2 = 10.35.

So z = 3.2 (1 d.p.)

Step-by-step explanation:

x=4.2

y=2.7

z=x

(×+y=×

(4.2+2.7=×)

×=6.9

55,555 times x = 965,777 teacher gave this weird question and i do not under stand help me please. also which pic is prettier be honest

Answers

Answer:

[tex]\frac{965777}{55555}[/tex]

Step-by-step explanation:

The answer is quite simple, just divide both sides and rewrite the equation to get it. Maybe focus more on the question rather than yourself next time :)

please solve this step by step I'll mark you as brainliest b(a-6)=

Answers

First you have to multiply each term by the parentheses by b so this would be it
ab-6b is the answer

Help me please. I need your help please.

Answers

Answer:

8/54 or 4/27

Step-by-step explanation:

(8/9)/6 =

8/9 ÷ 6 when divide turn the division sign into multiplication and flip the second fraction ( reciprocal))

8/9*1/6 = 8/54  if simplified =4/27

Answer:

4/27

Step-by-step explanation:

8/9 ÷ 6 = 4/27

Solve the system of inequalities: a) .4x−1≤0
2.3x≥4.6

Answers

Answer:

(a) [tex]x \leq 2.5[/tex]

(b) [tex]x \geq 2[/tex]

Step-by-step explanation:

Solving (a):

[tex]0.4x - 1 \leq 0[/tex]

Add 1 to both sides

[tex]0.4x - 1 + 1\leq 0 + 1[/tex]

[tex]0.4x \leq 1[/tex]

Solve for x

[tex]x \leq 1/0.4[/tex]

[tex]x \leq 2.5[/tex]

Solving (b):

[tex]2.3x \geq 4.6[/tex]

Solve for x

[tex]x \geq 4.6/2.3[/tex]

[tex]x \geq 2[/tex]

how do i solve for equations that have 2 variables

Answers

Answer:

To solve for an equation with two variables, substitute a zero(0) into the other variable and find one variable.

Step-by-step explanation:

To solve for an equation with two variables, substitute a zero(0) into the other variable and find one variable.

Simple Example:

3x+3y=12

Substitute 0 as y,

3x+0=12

3x=12

12/3=4

x=4

Now substitute 4 into the main equation,

3(4)+3y=12

12+3y=12

-12      -12

3y=0

0/3=0

y=0

So the answer is

3(4)+3(0)=12

12=12

To which place value is the number rounded?

8.293 to 8.29

a
ones
b
tenths
c
hundredths
d
thousandths

Answers

Answer:

c

Step-by-step explanation:  3 is not bigger than 9 so it can't bump it up so it stays like that.

show work
1) n - 8 = -3
2) -2m=-24
3) m/2=-7
4) 2/3y=20
(20 points for answers all)

Answers

Answer:

n = 5m = 12m= -14y =30

Step-by-step explanation:

1.

[tex]n -8 =-3\\\\Collect\:like\:terms\\n = -3+8\\\\Simplify ; -3+8 = 5\\\\n =5[/tex]

2.

[tex]-2m=-24\\\\Divide\: both \:sides\:of\:the\:equation\:by\:-2\\\\\frac{-2m}{-2} = \frac{-24}{-2} \\\\Simplify ; -2m/-2 = m , -24/-2 = 12\\\\m =12[/tex]

3.

[tex]\frac{m}{2}=-7\\\\Cross\:Multiply\\\\m\times 1 = 2\times -7\\\\m = -14[/tex]

4.

[tex]\frac{2}{3}y=20\\ \\\frac{2}{3} y = \frac{20}{1} \\\\Cross\:Multiply\\\\2y\times 1 = 3\times 20\\\\2y = 20\\\\\mathrm{Divide\:both\:sides\:by\:}2\\\frac{2y}{2}=\frac{60}{2}\\\\Simplify\\y = 30[/tex]

A is between B and C. BA = 11, BC = 20 find AC

Answers

Answer:

31

Step-by-step explanation:

Since be is in the middle its connecting the two.

So you will add the 11 and 20 to get 31

Answer :

31


Step By Step Equation

what number is 100 times greater than 20?

Answers

Answer:

2000

Step-by-step explanation:

just muliply

2000 is the correct answer

Question is -4x + 14 = 54

Answers

Answer: x = -10

Step-by-step explanation:

-4x + 14 = 54
-4x = 54 - 14
-4x = 40
x = 40/-4
x = -10

The diagram below shows
two similar
similar rectangles.
Find the perimeter of the
combined figure.
5 cm
13 cm​

Answers

Answer:

Perimeter of 1st rectangle=20cm

Perimeter of 2nd rectangle=52cm

If a line has a slope 2/3 a line that is perpendicular has a slope of what

Answers

Answer:

-1.5

Step-by-step explanation:

The product of slopes of two perpendicular lines = -1

If line one has a slope of 2/3;

Let line two, perpendicular to line one, has a slope of a

2/3 × a = -1

a = -1 ÷ 2/3 = -3/2 = -1.5

what is the domain of the function

Answers

Answer:

the domain or set of departure of a function is the set into which all of the input of the function is constrained to fall.

Step-by-step explanation:

didn't know if you needed the definition or not :)

The table above gives selected values of twice-differentiable functions f and g, as well as the first two derivatives of g. If f′ (x)=3 for all values of x, what is the value of [tex]\begin{equation}\int_{2}^{4} f(x) g^{\prime \prime}(x) d x\end{equation}[/tex]

Answers

Answer:

63

Step-by-step explanation:

∫₂⁴ f(x) g"(x) dx

Integrate by parts.

If u = f(x), then du = f'(x) dx.

If dv = g"(x) dx, then v = g'(x).

∫ u dv = uv − ∫ v du

= f(x) g'(x) − ∫ g'(x) f'(x) dx

f'(x) is constant at 3, so:

= f(x) g'(x) − ∫ 3 g'(x) dx

= f(x) g'(x) − 3 g(x)

Evaluate from x=2 to x=4.

[f(4) g'(4) − 3 g(4)] − [f(2) g'(2) − 3 g(2)]

[(13) (7) − 3 (9)] − [(7) (1) − 3 (2)]

64 − 1

63

The required value of [tex]\int\limits^4_2 {f(x)g"(x)} \, dx (1)[/tex] is 63.

Given that,

The table above gives selected values of twice-differentiable functions f and g,

The first two derivatives of g. If f′ (x) = 3 for all values of x.

We have to determine,

What is the value of [tex]\int\limits^4_2 {f(x)g"(x)} \, dx (1)[/tex].

According to the question,

To obtain the value of given function, integrating the function,

[tex]= \int\limits^4_2 {f(x)g"(x)} \, dx (1)[/tex]

Integrating by using by-parts.

[tex]= \int\limits^4_2 {f(x)g"(x)} \, dx (1)\\\\if\ u = f(x), \ then \ du = f'(x)dx\\\\if \ dv = g"(x)dx, \ then \ v = g'(x)\\\\[/tex]

Then,

[tex]\int u\ dv = uv\ - \ \int v \ du\\\\= f(x) \ g'(x) - \int g'(x) f'(x) \ dx\\[/tex]

if f'(x) is constant at 3,

Then,

[tex]= f(x) g'(x) - \int g'(x)\dx\\\\= f(x) g'(x) -3g(x)[/tex]

Substitute from x = 2 to x = 4 then evaluate,

[tex]= [f(4)\ g'(4) - 3 \ g (4)]- [f(2)\ g'(2) - 3 \ g (2)]\\\\= [(13).(7) - 3.(9)] - [7.(1)- 3.(2)]\\\\= [91-27} - {7-6}\\\\= 64-1\\\\= 63[/tex]

Hence, The required value of [tex]\int\limits^4_2 {f(x)g"(x)} \, dx (1)[/tex] is 63.

To know more about Integration click the link given below.

https://brainly.com/question/14716088


MONEY FOR FUN
If you earn $50,000 a year
and spend 5 percent of
that on sports and enter-
tainment, how much do
you spend per year on
sports and entertainment?
Th

Answers

Answer:

$2,500

Step-by-step explanation:

1.) 5% would be switched to decimal form which is 0.05

To do this, start with 5.

Move the the decimal point two place values to the left.

2.) Now multiply 50,000 and 0.05

3.) This equals 2,500

Lindsey started a savings account with $260. After 4 weeks, she has $315 dollars. After
8 weeks, she had $370. What is the rate of change of money in her savings account per
week?

Answers

Answer:

$13.75

Step-by-step explanation:

0 weeks = 260

4 weeks = 315

8 weeks = 370

Every 4 weeks, the rate of change is $55 as can be seen by 315 - 260 = 55 and 370 - 315 = 55.

Per week, the rate of change is $13.75 This is because $55 ÷ 4 weeks = rate of change per week

55 ÷ 4 = 13.75

Andrea wanted to know how much farther she ran on Tuesday than on Monday. Her work is below.

A student showing work for 4.705 minus 3.6 = 4.669. The decimal for 3.6 is between the 0 and 5 in 4.705.

What mistake did Andrea make when solving the problem?
She did not align the decimal points.
She did not rename correctly.
She added instead of subtracted.
She did not subtract 15 and 6 correctly.

Answers

Answer:

its A.

Step-by-step explanation:

I just answered it :)

Answer:

She did not align the decimal points.

Step-by-step explanation:

Julio is paid 1.4 times hiş normal hourly rate for each hour he works over 30 hours in a week. Last week he worked 35 hours and earned $436.60. Write and solve an equation to find Julio's normal hourly rate, r. Explain how you know that your answer is reasonable. *​

Answers

Answer:

Julio is paid 1.4 times his normal hourly rate for every excess hour he works in a week.

last week he worked 35 hours and got 436.60 dollars.

We need to find an equation so we can be able to solve Julio’s earnings:

=> Let X be the value of Julio’s rate

=> 30 * X + 5 x 1.4x = 436.6

=> 30x + 7x = 436.6

=> 37x = 436.6

=> 37x / 37 = 436.6 / 37

=> x = 11.8

Thus the original rate of Julio is 11.8 dollars

=> 1.4 x 11.8 = 16.52 dollars, the rate of Julio with added rate

=> 30 x 11.8  + 5 x 16.52

=> 354 + 82.6

=> 436.6

Step-by-step explanation:

A is the midpoint of BC.
BA = 3x+8
AC=23.
solve for x

Answers

Since A is the midpoint of BC, we can assume BA and AC are equal to each other.

We also know that BC = BA + AC = 23 + 23 = 46

Since A is the midpoint of BC, A divides BA and AC equally.
Therefore, BA=AC.
Since BA=AC,
3x+8=23
Or, 3x= 23-8
Or, 3x= 15
Or, x=15/3
Therefore, x=5.
Ans: x=5.

8 + 3a + 7 - 2a simplifies to

Answers

I’m pretty sure its 15+1a

8+7=15
3a-2a=1a

8+3a-2a+7=0

8+a+7=0

a+15=0

a= (-15)

Hope it helped

Thank You

Solve for d:
df + 5d = h

Answers

Answer:

d=hf+5

Step-by-step explanation:

Step 1: Factor out variable d.

d(f+5)=h

Step 2: Divide both sides by f+5.

d(f+5)f+5=hf+5

d=hf+5

Jasmine was observing her farm from a hot air ballon when she accidently dropped her telescope. As always on earth the formula to find the distance has fallen , d, after t seconds, is the triangle D= 16t^2. How far has the telescope fallen after 1 second. First person to answer gets brainliest

Answers

Answer:

D=16m

Step-by-step explanation:

D=16t^2

D=16×1

D=16m

I have an assignment due today. I need help.

If you have time to help me and your serious about it comment on this and I’ll send the problems I need help with.
Thank you

Answers

Answer:

62 m/s = 138.7 mi/h

Step-by-step explanation:

62 m/s = 62 x (3600/1000) x (1/1.61)

= 138.7 mi/h

Best Regards!

Multiply (−7)(2)(−2).

Answers

Answer:

28

Step-by-step explanation:

-7 × 2 = -14Plug -14: -14 × -2-14 × -2 = 28

I hope this helps!

Answer:

28!

Step-by-step explanation:

Which expressions have a sum of 12?

Answers

Answer:

First one.

Step-by-step explanation:

4+(-8) -10+22 =12

24+(-12) 0+ (-12) =-24

Please brainliest! Im on my way to another rank!

The expression 24 + (-12) has a sum of 12 after applying the arithmetic operation options (b) and (c) is correct.

What are arithmetic operations?

The study of numbers and their operations, which are relevant to all other fields of mathematics, is part of the discipline of arithmetic operations. Addition, subtraction, multiplication, and division are the basic operations that make up this system.

It is given that:

The expressions are shown in the picture.

After solving using Arithmetic operation:

Take the expression:

= 24 - 12

= 12

-10 + 22

= 12

Thus, options (b) and (c) are correct.

Learn more about the arithmetic operations here:

brainly.com/question/25834626

#SPJ2

find the value of [tex]\sqrt[3]{8}[/tex]

Answers

Answer:

∛8 = 2

Step-by-step explanation:

[tex]\sqrt[3]{8}[/tex]

=2

2*2*2 = 8

3y + 8 = ½ y + 28 Find y

Answers

[tex]\text{Hi there! :]}[/tex]

[tex]\large\boxed{y = 8}[/tex]

[tex]3y + 8 = 1/2y + 28\\\\\text{Subtract 8 from both sides:}\\\\3y + 8 - 8 = 1/2y + 28 - 8\\\\3y = 1/2y + 20\\\\\text{Subtract 1/2y from both sides:}\\\\3y - 1/2y = 1/2y - 1/2y+ 20\\ \\\text{Convert 3y to 6/2y to make subtracting easier:}\\5/2y = 20\\\\\text{Divide both sides by 5/2 (Multiply by reciprocal, or 2/5):}\\\\\\5/2y * 2/5 = 20 * 2/ 5\\\\y = 8[/tex]

what is the domain of f(x)=9-x²
A(.fx)≥9
B.All real numbers
C.-3≤x≤3
D. x≤9

Answers

Answer:

B. all real numbers

Step-by-step explanation:

hope that helps

Answer:

B  hope this helps :)

Step-by-step explanation:

Other Questions
8.3 + x= 12.1 I will love you if you can help me! how do you write 1:35 am spanish Which accident will be more damaging, collision between two trucksmoving with a speed 50 km/hr or collision between two cars moving with a speed of 50 km/hr? Explain. 768 pages in 8 days = _____ pages per day Do you think architects and developers now have a clearer picture of how the world should look compared to early developers? Why or why not? pls hurry10 pts + brainliest A city bus travels six blocks east and eight blocks north east wok is 100 m long is a bus travels as distance and 15 minutes what is the average speed on the bus A pendulum built from a steel sphere with radius r cm 5 and density stl kg m S 3 7800 is attached to an aluminum bar with length l m 1 thickness t cm 0 8. and width w cm 4 and density . al kg m S 3 2820 a. Calculate the mass moment of inertia of the pendulum about its center of mass, . cm I b. Calculate the mass moment of inertia of the pendulum about its pivot point, o I . Which of the following sentences is grammatically correct?a.Only Bill wanted to go to the lake. [Intended meaning: Bill wanted to go to the lake, but no one else did.]b.Bill only hinted that he would help us move. [Intended meaning: Bill hinted but did not, for example, promise that he would help us move.]c.Bill wanted only Mary to win. [Intended meaning; Bill wanted Mary to win, but did not want anyone else to win.d.all of these Read the passage from "Racing the Storm. Anastasia shook her head in wonder. "Youre so lucky you have your license, she said. "I wish I had been the one driving. The author develops Anastasias character through the dialogue. plot. mood. setting. . Popular music today is not as good as it was in the past. Fact or Opinion Explain: _____________________________________________________________ which two sentencens support the point that both cultures value children What 2 colors makengreen What is the depth of field?a. The area of the film in which light appears dark and dark appears light.b. The area of sharpest focus in a photograph.c. The area and time that the photograph is exposed to light.d. The area of a photograph produced by light falling on a light sensitive surface. I really need help with this before 5:10 Show your workYour big brother has been brageing about how he can bench press 4.000 grams at the gym. Sounds impressive.but you know better. Use your skills to find how many pounds of weight he is actually pressing. What do you think the story of Tom Sawyer and whitewashing the fence says about Tom as a manager? Would you call him a good manager or a bad manager? Why? PLZ HELP ME SOONNN Would the answer be 250 ? Your bedroom air conditioner blows very cold air at night but only cool air during the day. Your bedroom gets lots of direct sunlight all day long. What's the hypothesis and experiment. The ancient Egyptians used the practice of to cure certain illnesses. Rosa is putting carpet in her room. Shes using the scale 1 inch = 3 ft . In the drawing, the length is 5.5 inches and the width is 3.25 inches. If carpet costs 2.58 per square foot, how much will it cost to carpet the bedroom? I've tried soooo many times on this question, but I just can't get it! It's due later today, so if someone would please explain to me what I'm supposed to do. I know what to do, but I'm doing something wrong, but I don't know what. I need to write a recursive and explicit formula for the sequence -16, 4, -1, 1/4, -1/16, 1/64 It is a geometric sequence.